LSAT and Law School Admissions Forum

Get expert LSAT preparation and law school admissions advice from PowerScore Test Preparation.

 Administrator
PowerScore Staff
  • PowerScore Staff
  • Posts: 8916
  • Joined: Feb 02, 2011
|
#35167
Complete Question Explanation

Assumption. The correct answer choice is (E)

In this stimulus, the author argues against a hypothesis said to explain how the solar system was formed. That hypothesis asserts that a supernova created the cloud and dust that formed our solar system. The author points out that although supernovas produce the isotope iron-60, researchers have not found iron-60 in meteorites known to have been formed early in the history of the solar system. Based on this lack of iron-60 in early meteorites, the author concludes that the hypothesis is disproven, meaning that it is not true that the “solar system was formed from a cloud of gas and dust produced by a supernova.”

This conclusion is weak, because the author has made an assumption about the presence of iron-60 in meteorites. We accept as fact that supernovas produce iron-60. And we accept as fact that there are meteorites known to have formed early in the solar system’s history. But the author provides no factual basis for the position that meteorites formed early in the solar system’s history would contain iron-60.

This is an Assumption question. Since there is no new or “rogue” information in the conclusion, such as a term that was not introduced in a premise, this is a Defender style Assumption question. Our prephrase is that the correct answer choice will defend the conclusion by telling us that meteorites formed early in the solar system’s history would contain iron-60 if iron-60 was present in the solar system at that time.

Answer choice (A): This answer choice is confusingly worded, but it has no effect on the conclusion and is not required for the conclusion to be valid. Further, it may appear that this answer choice hurts the conclusion, by suggesting that the composition of early meteorites is not good evidence of the chemical elements present in the solar system at the time the meteorite was formed. However, that is not the case. The answer choice says that such meteorites “contain chemical elements that are unlikely to be found in gas and dust produced by a supernova.” However, it does not say that meteorites contain only such chemical elements. So, nothing in this answer choice is inconsistent with the author’s argument, though the answer is not required for the conclusion to be valid either.

Answer choice (B): This information about how other solar systems are formed is irrelevant to the conclusion, which was limited to a hypothesis concerning the formation of our solar system.

Answer choice (C): The correct answer choice in an Assumption question contains information logically required for the conclusion to be valid. That is a very restrictive standard to meet. It is irrelevant to this conclusion whether or not supernovas produce forms of iron other than iron-60. The author did not discuss the presence or absence of other forms of iron in the early meteorites, and did not address what the impact of the absence of such forms of iron would have on the hypothesis.

Answer choice (D): The time period involved in this answer choice, i.e., that period relatively late in the solar system’s history, makes this answer choice irrelevant to the conclusion, which dealt only with evidence from the early history of the solar system.

Answer choice (E): This is the correct answer choice, because it properly identifies the assumption made by the author concerning the presence of iron-60 in early meteorites. We can confirm that this is the correct answer choice by using the Assumption Negation Technique, by which we test the answer choice to determine if it is required for the conclusion. Once logically negated, the information in the correct answer choice will destroy the conclusion. Here, the logical negation of the answer choice is that iron-60 present in the early history of the solar system would not be found in meteorites formed early in the solar system’s history. If that negation is added to the stimulus as part of the argument, it destroys the conclusion. So, we can confirm that this is the correct answer choice.
 smm
  • Posts: 7
  • Joined: May 20, 2017
|
#38239
I've been looking at answer choice E for a while and it really looks like E justifies the conclusion but is not absolutely required for the conclusion. Here is the argument as I see it:

hypothesis: solar system was formed by gas cloud from supernova

conditional statement: hypothesis correct ----> iron-60 present in early solar system

premise: no iron-60 in early meteorites

conclusion: hypothesis is incorrect

To justify the conclusion you want to link "iron-60 in early solar system" to "iron-60 in early meteorites" as follows:

iron-60 in early solar system ----> iron-60 in early meteorites

E provides that conditional statement exactly. But is this statement required? The explanation above states that the the negation of answer choice E is "iron-60 present in the early history of the solar system would not be found in meteorites formed early in the solar system’s history"; however, I don't think that negation is correct. Shouldn't the negation be as follows?

iron-60 in early solar system --/--> iron-60 in early meteorites

in words: if iron-60 is in the early solar system it is not required that iron-60 be in early meteorites

So negating E states that if iron-60 is in the early solar system is is possible that it will not be in early meteorites. This negation certainly weakens the argument's conclusion but it doesn't seem to make it a logical impossibility. So, I fail to see how E is, in the strict sense of the word, "required".
User avatar
 Dave Killoran
PowerScore Staff
  • PowerScore Staff
  • Posts: 5853
  • Joined: Mar 25, 2011
|
#38260
Hi SMM,

Thanks for the question! This one provides us with a platform to talk about two very interesting points related to Justify and Assumption questions, so let's dive right in:

  • 1. The correct answer to an Assumption question (NA) could also be the correct answer to a Justify question (SA), and vice versa.

    There's a significant overlap between Justify and Assumption question, so your point here about this looking like an answer that Justifies actually has no impact on whether this is a correct answer in a straight Assumption question. Here's a simplified argument form that shows how that works:

    • Premise: A
      Conclusion: B

      Justify answer: A :arrow: B
      Assumption answer: A :arrow: B
    In the above, the answer to both question types is identical (although, as we know, we could add more to the Justify question answer and not make it incorrect, whereas doing so to the Assumption answer would). Thus, in this problem, while I agree that answer (E) helps the argument, that doesn't call it into question or make it wrong in the Assumption context.


    2. When negated, an Assumption of the argument need only hurt the argument, not totally destroy it.

    I personally think that negating (E) has a pretty powerful effect on the argument and puts it in serious jeopardy. It would make the author question the conclusion, because the absence of iron-60 is now longer conclusive proof that there was no supernova. But, let's say you read that in weaker terms, and felt the author would still have a comeback to the negation—would that rule out this answer? No, it would not. When we remove an assumption of an argument, we attack the foundation of the argument "building." However, there could be ways for the author to rush in and repair that foundation without the argument being utterly destroyed. So, the negation need only weaken or undermine the argument; it does NOT have to crush it completely (which is nearly impossible anyway in short form arguments such as these).

    The essence of Assumption Negation is to force the author to step back and have to repair the argument. If you crack the foundation, so to speak, you force the author to do just that. So, with the above negation, would the author ignore your attack, or be forced to deal with it? I don't think there's any question he or she would have to deal with it since the point is so central to the line of reasoning used.

    So, what about the "required" point you make at the end? Remember, when you negate something, you turn it into a different question type entirely (Weaken), so it's easy to think that the "required" element isn't there when evaluating it from that new perspective. I'd say here this is very clearly required by the argument in it's original form, but I think negating it caused you to attempt to evaluate in from a different standard (Weaken) than that used in Assumptions (Required), hence the confusion.

Both the above points are very high level, and so you can see you've crossed into some very interesting territory. Congrats to you on looking at this problem this closely—I'm impressed!
 smm
  • Posts: 7
  • Joined: May 20, 2017
|
#38334
Great answer (especially the part about attacking the arguments foundation)! This question makes sense now. I was getting too hung up on wanting the negation of E to make the argument's conclusion impossible. Thanks!

Get the most out of your LSAT Prep Plus subscription.

Analyze and track your performance with our Testing and Analytics Package.